Possible Error in LSAT 64, Section 3, Question 25 Forum

Prepare for the LSAT or discuss it with others in this forum.
User avatar
LSAT Hacks (Graeme)

Bronze
Posts: 371
Joined: Wed May 30, 2012 9:18 pm

Possible Error in LSAT 64, Section 3, Question 25

Post by LSAT Hacks (Graeme) » Mon May 13, 2013 8:51 pm

This is the question about trade union members. I believe that the stimulus has a flawed argument, while the correct answer has a good argument. I may write to the LSAT about this, but I wanted to post here first to make sure I'm not hallucinating. (The latter is more likely)

----------------------------
Argument

"Independent arbitration *would* avert a strike."

That is a sufficient condition. Arbitration --> ~Strike

The argument then says that union support is necessary for arbitration, and the union likely won't support arbitration.

Arbitration --> Union Support

This is enough to conclude that there won't be arbitration. But I don't see how we can conclude a strike is unlikely, because it might be avoided for another reason. Maybe management will pay the union's demands, maybe the government will intervene, etc.

In my opinion, this argument incorrectly assumes that lack of a sufficient condition negates a necessary condition.

---------------------
Correct Answer

The credited answer is D. In that argument, Lopez wins only if he is hydrated:

Win --> hydrated

His team probably won't hydrate him. So he probably won't win. This is a good argument.

--------------------------------------

If my analysis above is correct, then the argument's aren't really parallel. They're close, but the stimulus makes an error while the correct answer doesn't. That seems a stretch for parallel reasoning. So, two questions:

1. Am I right that the argument in the stimulus is flawed?
2. Is the argument in answer choice D good?
3. If 1 and 2 are true, then can D justifiably be the correct answer for a parallel reasoning question?

If I'm wrong about the stimulus, please let me know. I'm doubly confused because I WROTE explanations for this test and for this question (they're on sale on LSAT Blog). In those explanations I said the argument in the stimulus was correct.

So past me disagrees with me. Who's right?

Daily_Double

Silver
Posts: 1031
Joined: Tue Dec 04, 2012 8:45 pm

Re: Possible Error in LSAT 64, Section 3, Question 25

Post by Daily_Double » Mon May 13, 2013 9:37 pm

Ok. So first I'm going to do the question, then I'll try to account for your issues.

PT 64, S3, Q25: Parallel

Stimulus:

Action is planned
NOT A ---> B
B is unlikely to happen, based upon past
____________________________________
TH: A is likely

First Pass:
A: First premise is too strong, second premise provides a sufficient condition instead of the negated necessary condition, with some uncertainty. Conclusion is too strong. And this is a perfect conditional argument. We're not looking for conditional linkage in this form. We need a bit of uncertainty.

B: God no. This argument is severely flawed.This argument assumed that if a condition necessary for a condition is not prevented from occurring, then the a condition sufficient for that condition will occur.

C: God no. This argument fails to justify the conclusion and isn't structurally the same as the stimulus.

D: This looks good. Here we have:

Action is planned.
If A ---> B
B is unlikely to happen, based on past
_________________________________
TH: NOT A is probable

E: This answer has a mistaken negation. Eliminate.

And that's it. We found it on the first pass. Now let's see what you think:
graeme wrote:Argument

"Independent arbitration *would* avert a strike."

That is a sufficient condition. Arbitration --> ~Strike

The argument then says that union support is necessary for arbitration, and the union likely won't support arbitration.

Arbitration --> Union Support

This is enough to conclude that there won't be arbitration. But I don't see how we can conclude a strike is unlikely, because it might be avoided for another reason. Maybe management will pay the union's demands, maybe the government will intervene, etc.

In my opinion, this argument incorrectly assumes that lack of a sufficient condition negates a necessary condition.
I think you're thinking too hard. The stimulus goes:

(p1) Action is planned
(p2) If Action doesn't happen ---> Agreement
(p3) Agreement is unlikely, based upon past
___________________________________
TH: Action is likely

What the argument does is take the contrapositive of (p2) with the addition of a bit of uncertainty. The only possible flaw here would be a temporal one, because the argument infers something is of the future, based upon the past. However, I think the argument covers its bases with the addition of the uncertainty.
graeme wrote:1. Am I right that the argument in the stimulus is flawed?
2. Is the argument in answer choice D good?
3. If 1 and 2 are true, then can D justifiably be the correct answer for a parallel reasoning question?
1. No, in my opinion you are not correct. However, I could be mistaken.
2. Yes, it's very good
3. To begin with, I haven't satisfied your sufficient condition. But, what you're asking here is that if we have a flawed stimulus of the parallel question type, then can the correct answer not be flawed. The answer to that question is no. But I've accounted for that above, both are valid.

EDIT: Wait, I think I see what you're getting at. Basically, in the stimulus, to describe in more detail: we have a conclusion that an action is likely to happen solely because one think that would prevent that action will not occur. But we don't know that's the only thing.

Ahhh, great point. I think you're on to something here. But I still think that the uncertainty accounts for it.

User avatar
LSAT Hacks (Graeme)

Bronze
Posts: 371
Joined: Wed May 30, 2012 9:18 pm

Re: Possible Error in LSAT 64, Section 3, Question 25

Post by LSAT Hacks (Graeme) » Mon May 13, 2013 10:12 pm

Yeah, your edit catches my central point. I think arbitration was sufficient to avert the strike, but they treated it as necessary. So the stimulus is flawed, making a classic error of logic.

There's nothing wrong with the probabilistic nature of the conclusion. Actually, it was nicely paralleled in the credited answer.

However, I can't recall any other question where the stimulus had a bad argument and the right answer had a good argument. It seems too large a structural gap.

User avatar
wtrc

Gold
Posts: 2053
Joined: Mon May 30, 2011 9:37 pm

Re: Possible Error in LSAT 64, Section 3, Question 25

Post by wtrc » Mon May 13, 2013 10:39 pm

This question seemed to be an easy one when taking the LSAT, because the other 4 answer choices sucked. But looking at it now, I think you are right. We cannot conclude a strike is likely only because the union is "quite unlikely to agree to this," unless the stimulus said "ONLY independent arbitration would avert a strike."

Answer choice D uses the word "only," and thus is correct reasoning.

I've never seen a flawed stim have the correct answer choice be good logic. I've also never seen a flawed stim with a question stem that doesn't say "flawed pattern of reasoning." You just owned LSAC.

User avatar
LSAT Hacks (Graeme)

Bronze
Posts: 371
Joined: Wed May 30, 2012 9:18 pm

Re: Possible Error in LSAT 64, Section 3, Question 25

Post by LSAT Hacks (Graeme) » Mon May 13, 2013 11:11 pm

Cool. I've written LSAC to argue that the question is flawed.

We shall see. I wrote in once before, because I thought a second answer was better. I received a three page reply that convinced me I was wrong. Still makes me smile thinking of it.

Want to continue reading?

Register now to search topics and post comments!

Absolutely FREE!


Daily_Double

Silver
Posts: 1031
Joined: Tue Dec 04, 2012 8:45 pm

Re: Possible Error in LSAT 64, Section 3, Question 25

Post by Daily_Double » Mon May 13, 2013 11:13 pm

graeme wrote:Yeah, your edit catches my central point. I think arbitration was sufficient to avert the strike, but they treated it as necessary. So the stimulus is flawed, making a classic error of logic.

There's nothing wrong with the probabilistic nature of the conclusion. Actually, it was nicely paralleled in the credited answer.

However, I can't recall any other question where the stimulus had a bad argument and the right answer had a good argument. It seems too large a structural gap.
I think the flaw would really be takes the absence of a condition which would make an action unlikely, to infer that the action would be likely. Or, alternatively, assumes, without providing justification, that there do not exist other means to prevent a strike which are likely. Alternatively, the argument fails to take into account other means which are both likely and would prevent a strike. But this is really just rewording your view of the flaw, since we can also interpret the stimulus into a separate conditional relationships:

Arbit ---> ~Strike
Abit would prevent strike ---> Agreement
~Agreement

Now the tricky part. What we can infer here is that arbitration would not prevent a strike. The argument infers from this that a strike is probable. So the argument takes the absence of a condition which would not result in an action to infer the presence of the action.

At issue here is really whether the removal of an action which would prevent something from taking place makes it likely the action will take place. I think you've done a great job here reading between the lines when most testers, myself included since I have taken this one and didn't notice this one was flawed, would simply match the structure, recognize the others are obviously wrong and move onto question 26. If the argument simply added the word more into the conclusion, preceding the word likely, then it would work.
graeme wrote:1. Am I right that the argument in the stimulus is flawed?
2. Is the argument in answer choice D good?
3. If 1 and 2 are true, then can D justifiably be the correct answer for a parallel reasoning question?
So I take back what I said earlier. It would seem the correct answers to your questions are:

1. Yes
2. Yes, because it's the only one which roughly matches the stimulus.
3. Apparently so. Though, I have to admit, I'm pretty unhappy with this one as well. Of course, even if you did speak to LSAC, they would quote the question stem and say that answer choice D exhibits a pattern... most similar. But good luck.

Keep us in the loop, because this is odd.

User avatar
LSAT Hacks (Graeme)

Bronze
Posts: 371
Joined: Wed May 30, 2012 9:18 pm

Re: Possible Error in LSAT 64, Section 3, Question 25

Post by LSAT Hacks (Graeme) » Mon May 13, 2013 11:22 pm

Yeah, you're right that they could fall back on it being 'most similar', because it definitely is. I'd say they're violating a convention of the test, but not a rule.

I'll keep you guys posted. I appreciate the detailed commentary. When I disagree with an LSAT question, my first instinct is to suspect I'm missing something.

I only noticed this because I was trying to explain it to a student, and couldn't. I had spent a couple minutes drawing the stimulus and explaining that it was flawed, then found nothing flawed and similar in the answers....

(On a timed test I'm sure I would have picked D and moved on, as none of the other answers were even close)

User avatar
drawstring

Gold
Posts: 1933
Joined: Tue Apr 09, 2013 4:52 pm

Re: Possible Error in LSAT 64, Section 3, Question 25

Post by drawstring » Mon May 13, 2013 11:46 pm

Nice catch.

The reasoning in the stimulus is definitely flawed, which I didn't notice when previously doing the question.

User avatar
Micdiddy

Gold
Posts: 2231
Joined: Tue Mar 20, 2012 3:38 pm

Re: Possible Error in LSAT 64, Section 3, Question 25

Post by Micdiddy » Tue May 14, 2013 3:00 am

Yeah, the stim is definitely flawed. It should say something like "The strike can be averted only if..." but as worded all we can concluded is that independent arbitration will not avert the strike. Keep us informed on how LSAC responds!

Want to continue reading?

Register for access!

Did I mention it was FREE ?


User avatar
Clearly

Gold
Posts: 4189
Joined: Sat Feb 11, 2012 4:09 pm

Re: Possible Error in LSAT 64, Section 3, Question 25

Post by Clearly » Tue May 14, 2013 4:08 am

I thought for sure you were gonna be messing something up (I can't tell you how many times I've thought a question was wrong, only to go back say ah shit, I see now)...But having looked at the question now, you're dead on in this case.

I guess the question becomes does as parallel reasoning question committing a minor flaw have to be labeled as a parallel flaw question? Technically the answer is still D, as the "pattern of reasoning is most similar to" it even if one is flawed and one isn't...still vastly more similar to it than any other answer choice; so while its certainly weird, I can't say it's technically wrong.

User avatar
Gamine

New
Posts: 89
Joined: Mon Mar 18, 2013 6:03 am

Re: Possible Error in LSAT 64, Section 3, Question 25

Post by Gamine » Tue May 14, 2013 7:59 am

graeme wrote:Cool. I've written LSAC to argue that the question is flawed.

We shall see. I wrote in once before, because I thought a second answer was better. I received a three page reply that convinced me I was wrong. Still makes me smile thinking of it.
Wooow...that's so cool

bp shinners

Gold
Posts: 3086
Joined: Wed Mar 16, 2011 7:05 pm

Re: Possible Error in LSAT 64, Section 3, Question 25

Post by bp shinners » Tue May 14, 2013 11:38 am

Clearlynotstefan wrote:I guess the question becomes does as parallel reasoning question committing a minor flaw have to be labeled as a parallel flaw question? Technically the answer is still D, as the "pattern of reasoning is most similar to" it even if one is flawed and one isn't...still vastly more similar to it than any other answer choice; so while its certainly weird, I can't say it's technically wrong.
I wouldn't characterize this as a minor flaw.

And to answer your question, no, it doesn't. There are plenty of Parallel questions out there with flaws that aren't labeled as Parallel Flaw questions - in that case, the overall structure of the argument AND the flaw both needed to be parallel (as opposed to a Parallel Flaw question, where just the flaw is mirrored).

So again, while not technically an issue here if you read just the text of the question, it does violate the conventions of the test that the LSAC has established - if you say a flawed argument is parallel to a valid argument, you've got issues with the question type. (I haven't checked out the question - this is assuming Graeme/Stefan are correct in their assessment, and I have respect for both so I'm willing to go with it.)

melmoththewanderer

New
Posts: 86
Joined: Sun Apr 14, 2013 12:31 pm

Re: Possible Error in LSAT 64, Section 3, Question 25

Post by melmoththewanderer » Tue May 14, 2013 6:37 pm

I've looked at this and want to suggest something.

I think they have something different in mind here, and here is how I diagrammed it.

Independent arbitration (IA) is sufficient to avert a strike (AS) only if both sides agree (BSA)

(IA --> AS) --> BSA

The contrapositive is:

~BSA --> ~(IA --> AS)

It is given that both sides are unlikely to agree and because there exists an arbitrator's recommendation, we can grant independent arbitration has occurred. By way of the contrapositive, we see that it is not sufficient to avert strike.

So, to answer your question, my verdict is this question is not flawed.
Last edited by melmoththewanderer on Tue May 14, 2013 6:42 pm, edited 1 time in total.

Register now!

Resources to assist law school applicants, students & graduates.

It's still FREE!


User avatar
CardozoLaw09

Gold
Posts: 2232
Joined: Sat Aug 28, 2010 1:58 pm

Re: Possible Error in LSAT 64, Section 3, Question 25

Post by CardozoLaw09 » Tue May 14, 2013 6:41 pm

melmoththewanderer wrote:I've looked at this and want to suggest something.

I think they have something different in mind here, and here is how I diagrammed it.

Independent arbitration (IA) is sufficient to avert a strike (AS) only if both sides agree (BSA)

(IA --> AS) --> BSA

The contrapositive is:

~BSA --> ~(IA --> AS)

Because there exists an arbitrator's recommendation, we can grant independent arbitration has occurred. By the contrapositive, we see that it is not sufficient to avert strike.

So, to answer your question, my verdict is this question is not flawed.
This is how I saw it as well

Daily_Double

Silver
Posts: 1031
Joined: Tue Dec 04, 2012 8:45 pm

Re: Possible Error in LSAT 64, Section 3, Question 25

Post by Daily_Double » Tue May 14, 2013 6:55 pm

I'm going to address this in parts. So first off, your theory, which I agree with, in part:
melmoththewanderer wrote: Independent arbitration (IA) is sufficient to avert a strike (AS) only if both sides agree (BSA)

(IA --> AS) --> BSA

The contrapositive is:

~BSA --> ~(IA --> AS)

It is given that both sides are unlikely to agree and because there exists an arbitrator's recommendation, we can grant independent arbitration has occurred. By way of the contrapositive, we see that it is not sufficient to avert strike.
What can infer here is that Independent arbitration is not likely to be sufficient to avert a strike. But that doesn't mean that a strike is likely. It just means that the prevention of a strike is less likely.
melmoththewanderer wrote:So, to answer your question, my verdict is this question is not flawed.
The issue here, which I tried to go into a bit of detail in my second post, is that the argument assumes that, based upon the likely absence of one obstacle which would prevent an event from taking place that the event itself is likely to take place. Do you see what I'm getting at here? To use an example from the movie 300, the removal of the crippled guy would prevent the Persians from discovering a passage which was sufficient to allow them to win the war against the Spartans. But we can't infer that because the Persians can't win the war using that method, that the Spartans are likely to win the war. We can infer that they are less likely to lose, since one measure which was sufficient for them to lose the war will not take place.

Do you follow?
Last edited by Daily_Double on Tue May 14, 2013 7:00 pm, edited 1 time in total.

User avatar
drawstring

Gold
Posts: 1933
Joined: Tue Apr 09, 2013 4:52 pm

Re: Possible Error in LSAT 64, Section 3, Question 25

Post by drawstring » Tue May 14, 2013 6:57 pm

melmoththewanderer wrote:I've looked at this and want to suggest something.

I think they have something different in mind here, and here is how I diagrammed it.

Independent arbitration (IA) is sufficient to avert a strike (AS) only if both sides agree (BSA)

(IA --> AS) --> BSA

The contrapositive is:

~BSA --> ~(IA --> AS)

It is given that both sides are unlikely to agree and because there exists an arbitrator's recommendation, we can grant independent arbitration has occurred. By way of the contrapositive, we see that it is not sufficient to avert strike.

So, to answer your question, my verdict is this question is not flawed.
But doesn't that just suggest that it's unlikely a strike will be averted through independent arbitration, which hasn't been deemed necessary to avert a strike?

Given that many factors could influence the likelihood of a strike's occurrence, isn't it flawed to make the general claim that a strike is likely because it probably won't be averted through IA?

User avatar
LSAT Hacks (Graeme)

Bronze
Posts: 371
Joined: Wed May 30, 2012 9:18 pm

Re: Possible Error in LSAT 64, Section 3, Question 25

Post by LSAT Hacks (Graeme) » Tue May 14, 2013 7:36 pm

drawstring wrote:
But doesn't that just suggest that it's unlikely a strike will be averted through independent arbitration, which hasn't been deemed necessary to avert a strike?

Given that many factors could influence the likelihood of a strike's occurrence, isn't it flawed to make the general claim that a strike is likely because it probably won't be averted through IA?
This was my conclusion. The strike could be averted by the government, by a management offer, by etc.

Get unlimited access to all forums and topics

Register now!

I'm pretty sure I told you it's FREE...


melmoththewanderer

New
Posts: 86
Joined: Sun Apr 14, 2013 12:31 pm

Re: Possible Error in LSAT 64, Section 3, Question 25

Post by melmoththewanderer » Tue May 14, 2013 8:28 pm

Daily_Double wrote:I'm going to address this in parts. So first off, your theory, which I agree with, in part:
melmoththewanderer wrote: Independent arbitration (IA) is sufficient to avert a strike (AS) only if both sides agree (BSA)

(IA --> AS) --> BSA

The contrapositive is:

~BSA --> ~(IA --> AS)

It is given that both sides are unlikely to agree and because there exists an arbitrator's recommendation, we can grant independent arbitration has occurred. By way of the contrapositive, we see that it is not sufficient to avert strike.
What can infer here is that Independent arbitration is not likely to be sufficient to avert a strike. But that doesn't mean that a strike is likely. It just means that the prevention of a strike is less likely.
melmoththewanderer wrote:So, to answer your question, my verdict is this question is not flawed.
The issue here, which I tried to go into a bit of detail in my second post, is that the argument assumes that, based upon the likely absence of one obstacle which would prevent an event from taking place that the event itself is likely to take place. Do you see what I'm getting at here? To use an example from the movie 300, the removal of the crippled guy would prevent the Persians from discovering a passage which was sufficient to allow them to win the war against the Spartans. But we can't infer that because the Persians can't win the war using that method, that the Spartans are likely to win the war. We can infer that they are less likely to lose, since one measure which was sufficient for them to lose the war will not take place.

Do you follow?
Excellent points being made here!

To respond, I want to point out that the conclusion is qualified by drawing an analogy "based on past experience." So we do have to take into account that the facts he's given us is he's qualifying his conclusion that all things between past and present are equal. This becomes the grounds for his conclusion that the strike is likely. There's quite a bit of leeway we can grant in the qualified conclusion, but I'd venture to say that if what you have (based on past experience--i.e. assuming the past and present are analogous) is not sufficient to avert the strike, then the strike is likely.

Daily_Double

Silver
Posts: 1031
Joined: Tue Dec 04, 2012 8:45 pm

Re: Possible Error in LSAT 64, Section 3, Question 25

Post by Daily_Double » Tue May 14, 2013 9:35 pm

melmoththewanderer wrote: To respond, I want to point out that the conclusion is qualified by drawing an analogy "based on past experience." So we do have to take into account that the facts he's given us is he's qualifying his conclusion that all things between past and present are equal. This becomes the grounds for his conclusion that the strike is likely. There's quite a bit of leeway we can grant in the qualified conclusion, but I'd venture to say that if what you have (based on past experience--i.e. assuming the past and present are analogous) is not sufficient to avert the strike, then the strike is likely.
Huh, I don't see the phrase as having that characteristic. The characteristic being that it holds all else equal. And even if that were true, so we'd have, the absence of one obstacle to a strike, the conclusion would read more likely, or less unlikely. By stating that the action is likely to take place, it assumes that there do not exist significant amounts of other obstacles that are likely. I the phrase read the phrase to basically say: at least one event in the past leads this journalist to infer that the union will probably not agree to this.

Then the argument proceeds to make an invalid inference: because of the likely absence of a condition necessary for an for an event to prevent an action from occurring, that the action will likely occur. When all it has proved, to use the language above, is that this event will probably not prevent an action from occurring. But so what? That's one event. Maybe hundreds of others exist. I just don't really see how that phrase changes anything besides establishing that since the necessary condition will likely not occur, the sufficient condition will probably not occur.

melmoththewanderer

New
Posts: 86
Joined: Sun Apr 14, 2013 12:31 pm

Re: Possible Error in LSAT 64, Section 3, Question 25

Post by melmoththewanderer » Tue May 14, 2013 10:11 pm

Daily_Double wrote:
melmoththewanderer wrote: To respond, I want to point out that the conclusion is qualified by drawing an analogy "based on past experience." So we do have to take into account that the facts he's given us is he's qualifying his conclusion that all things between past and present are equal. This becomes the grounds for his conclusion that the strike is likely. There's quite a bit of leeway we can grant in the qualified conclusion, but I'd venture to say that if what you have (based on past experience--i.e. assuming the past and present are analogous) is not sufficient to avert the strike, then the strike is likely.
Huh, I don't see the phrase as having that characteristic. The characteristic being that it holds all else equal. And even if that were true, so we'd have, the absence of one obstacle to a strike, the conclusion would read more likely, or less unlikely. By stating that the action is likely to take place, it assumes that there do not exist significant amounts of other obstacles that are likely. I the phrase read the phrase to basically say: at least one event in the past leads this journalist to infer that the union will probably not agree to this.

Then the argument proceeds to make an invalid inference: because of the likely absence of a condition necessary for an for an event to prevent an action from occurring, that the action will likely occur. When all it has proved, to use the language above, is that this event will probably not prevent an action from occurring. But so what? That's one event. Maybe hundreds of others exist. I just don't really see how that phrase changes anything besides establishing that since the necessary condition will likely not occur, the sufficient condition will probably not occur.
Thanks for prodding this along. Let me try to address your claims. So the argument in the stimulus appeals to analogy here, which is a valid rhetorical move for inductive reasoning, which is the standard dealing with probabilities. It is a reasonable to assume, when the argument is appealing to an analogy between past and present that those cases be relevantly similar. So, my point is because it is given (by the phrase "based on past experience") that this sufficiently applies when the past and present are relevantly similar, we have to account for this fact in our interpretation of the stimulus. So if they aren't sufficiently similar, it is out of the argument's scope.

To address your second claim, of course there could be numerous other z-factors. But we can't assume they do or don't exist, because it isn't given in the stimulus. I still think the standard of reasonable assumptions prevails here. Sure you can make up your own premise and say that if Mariah Carey shows up the strike will be averted and then assume that Mariah Carey does in fact show up and conclude the entire question is bunk, since there will be no strike. But that involves 2 new assumptions that you can multiply ad infinitum with whatever permutation of examples.

My other point is that the conclusion says based on (1) appeal to past (and its reasonable assumptions) and (2) the fact that the union is unlikely to agree on this. So if you're adding another z-factor, then you are in effect adding another premise that is out of the argument's scope. Its no longer based on (1) and (2) but rather based on (1) and (2) and (z).

Daily_Double

Silver
Posts: 1031
Joined: Tue Dec 04, 2012 8:45 pm

Re: Possible Error in LSAT 64, Section 3, Question 25

Post by Daily_Double » Tue May 14, 2013 10:48 pm

melmoththewanderer, correct me if I'm misinterpreting your claim here. But it would appear that your are of the opinion that the question stimulus is not flawed because of this appeal to the past and the implicit analogy of that appeal. I'm saying this because without that appeal, which means the corresponding premise simply states: agreement unlikely and if we look at this modified argument from a structural point of view, then it is truly flawed. Therefore since you maintain that the argument with the appeal is valid, I can only conclude that this appeal is a primary reason you believe it to be so.

I'm just saying that the stimulus is flawed, mainly because I don't think the appeal to the past, which is something I see as a means to justify the probability of the occurrence of an action in the present and nothing more, makes the argument any less invalid. Though I will say that I have never thought about the statement "based upon the past" as anything more than my characterization of it above, nor have I come across anything that would lead me to believe otherwise, excluding your explanation, so I suppose I have some reading to do because I thought I had at least a strong foundational knowledge of everything on this test. Any recommendations?

Communicate now with those who not only know what a legal education is, but can offer you worthy advice and commentary as you complete the three most educational, yet challenging years of your law related post graduate life.

Register now, it's still FREE!


User avatar
drawstring

Gold
Posts: 1933
Joined: Tue Apr 09, 2013 4:52 pm

Re: Possible Error in LSAT 64, Section 3, Question 25

Post by drawstring » Tue May 14, 2013 11:11 pm

melmoththewanderer wrote:To address your second claim, of course there could be numerous other z-factors. But we can't assume they do or don't exist, because it isn't given in the stimulus. I still think the standard of reasonable assumptions prevails here. Sure you can make up your own premise and say that if Mariah Carey shows up the strike will be averted and then assume that Mariah Carey does in fact show up and conclude the entire question is bunk, since there will be no strike. But that involves 2 new assumptions that you can multiply ad infinitum with whatever permutation of examples.
Those factors aren't listed in the stimulus, but the argument explicitly gives a sufficient and not a necessary condition, then concludes that something is likely because the sufficient condition isn't met. Arriving at such a conclusion based on a sufficient condition not occurring is fallacious, whether or not we assume Mariah Carey shows up, and I'm certain there are flaw questions in which this is supposed to be identified as the flaw committed.

It would be like saying: "Drawstring won't join the circus if he's paid $100,000. Based on previous comparable situations, no one will pay him $100,000, so he'll likely join circus." Do you consider this flaw-free reasoning? I'd guess that you don't, even though it concludes that something is likely simply because a sufficient condition for it not occurring wasn't met.
the argument in the stimulus appeals to analogy here, which is a valid rhetorical move for inductive reasoning, which is the standard dealing with probabilities. It is a reasonable to assume, when the argument is appealing to an analogy between past and present that those cases be relevantly similar. So, my point is because it is given (by the phrase "based on past experience") that this sufficiently applies when the past and present are relevantly similar, we have to account for this fact in our interpretation of the stimulus.
The analogy concerns only independent arbitration though. Since IA is referred to as a sufficient condition but not referred to as a necessary condition, I don't see how it's unreasonable to assume that it's not a necessary condition. For the reasoning to be valid, doesn't one also have to assume that it's a necessary condition? That's the only way for other potential solutions to be irrelevant if the issue is not resolved through IA.

melmoththewanderer

New
Posts: 86
Joined: Sun Apr 14, 2013 12:31 pm

Re: Possible Error in LSAT 64, Section 3, Question 25

Post by melmoththewanderer » Tue May 14, 2013 11:48 pm

Let me put this in conceptual terms.

So there are a few things going on here.

First, we have a necessary condition of a conditional.

Second is a conditional conclusion. That is based on the fact that (given the fact, suppose, if) we are likely to fail the necessary condition of a conditional, we are likely to fail the conditional. So I think what we're missing here is that argument is calculating probability for likely failing a necessary condition of a conditional statement.

[let ~ mean not likely

(A-->B) --> C
~C --> ~(A-->B)

If you try to introduce a Z factor, it is out of the argument's scope.

It would no longer be strictly speaking

(A-->B) --> C
~C --> ~(A-->B)

but rather

(A-->B) --> C
z-->~B
~C + z --> ~(A-->B)

In which case, the conditional conclusion does not apply. I'm not saying that you have to assume only ~C, but rather that ~C + z factor is out of scope of this qualified conclusion, which concerns when C is unlikely. You can't, based on information given, calculate a probability for unlikely C plus z factor, and its not assumed that unlikely C would be sufficient under those conditions for there to be a strike.

So to answer y'alls questions, it is the difference between calculating probability for likely failing a necessary condition of a conditional statement and calculating the probability for likely failing a necessary condition of a conditional plus introducing a z factor. They aren't the same probabilities and because the conclusion is not strictly speaking a guarantee, they aren't assumed to be similar cases.

Daily_Double

Silver
Posts: 1031
Joined: Tue Dec 04, 2012 8:45 pm

Re: Possible Error in LSAT 64, Section 3, Question 25

Post by Daily_Double » Wed May 15, 2013 12:32 am

I think I follow you here, but I'm still not convinced, call it stubborness or maybe just because I'm convinced I'm correct here in saying the argument is flawed and I usually stick by my reasoning. But by your line of reasoning, then the following argument is not flawed:

Daenerys is planning to rule the seven kingdoms. The Lannisters would prevent this action, but only if they are not burned alive by dragons. Based upon the past, they will likely be burned alive by dragons, so Daenerys is likely to rule the seven kingdoms.

I'm hoping that a parallel argument will help you see what I'm getting at. Because all I'm trying to do is point out that the argument in the stimulus is flawed, and we seem to disagree.

So in both the example above and the stimulus we have:

Action is planned to take place in the future
If an event happens then the action would be prevented ---> necessary condition
The necessary condition will probably not occur, based on past
________________________________________________
The action will probably take place in the future

Logically, we can use the third premise to negate the conditional sufficient premise in the second premise, which translates to: the action would not be prevented. So in HBO fueled argument above, we can conclude only that the Lannisters would not likely prevent this action. But we have simply removed one obstacle to the action stated in the conclusion. Are there other obstacles that are would prevent this action? Do you see what I'm getting at? The argument fails to show that this claimed condition which would prevent an action is the only one that is likely, there can be many more. I mean, it's not necessary for it to show there is only one, but by only taking away one obstacle, the argument has a gap. For example, using the argument above, what if other houses would likely prevent Daenerys from ruling the seven kingdoms? The argument concludes that she likely will in fact rule them, but it hasn't addressed any other obstacles.
melmoththewanderer wrote:If you try to introduce a Z factor, it is out of the argument's scope.

It would no longer be strictly speaking

(A-->B) --> C
~C --> ~(A-->B)
I don't exactly follow. In both the stimulus and the example above, I'm not introducing another factor, I'm simply pointing out that the argument fails to justify the conclusion. It has however eliminated one possible obstacle to the occurrence of an action.
melmoththewanderer wrote:They aren't the same probabilities and because the conclusion is not strictly speaking a guarantee, they aren't assumed to be similar cases.
Good point, however the conclusion goes too far in my opinion because it fails to show that the occurrence of the action is in fact likely. I'll grant that it is more likely, but it still fails to show that the action will probably take place.

User avatar
Micdiddy

Gold
Posts: 2231
Joined: Tue Mar 20, 2012 3:38 pm

Re: Possible Error in LSAT 64, Section 3, Question 25

Post by Micdiddy » Wed May 15, 2013 4:24 am

@melmoth: we have no need to assume that other things can avert the strike in order to call it flawed. The fact that the stimulus did not rule out this possibility is enough to call it flawed, without stretching so far as to assume anything. It's a mistaken negation, period.

Also, to address another point, I would be extremely surprised if the LSAC comes back and says "a flawed argument can be paralleled with a good one." I simply don't believe they would ever venture to make such a statement. This answer is NOT more right than any other answer. The contrapositive and the mistaken negation, even with quantifiers thrown in, cannot be misconstrued to parallel each other anymore than a raven and a writing desk. Either the LSAC will explain where we were mistaken, or admit they did something wrong (or just not respond).

Seriously? What are you waiting for?

Now there's a charge.
Just kidding ... it's still FREE!


Post Reply

Return to “LSAT Prep and Discussion Forum”